LSAT and Law School Admissions Forum

Get expert LSAT preparation and law school admissions advice from PowerScore Test Preparation.

 Administrator
PowerScore Staff
  • PowerScore Staff
  • Posts: 8916
  • Joined: Feb 02, 2011
|
#40712
Complete Question Explanation
(The complete setup for this game can be found here: lsat/viewtopic.php?t=3641)

The correct answer choice is (E)

Ominously, this question stem is very similar to that of the prior question (a variable is placed fourth). And regrettably, this question turns out to be particularly brutal.

If G is fourth, then from the fourth rule J must be first, second, or third, and R must be fifth, sixth, or seventh.

From the original Not Laws, Y cannot be among the first three articles edited, and thus Y must be fifth, sixth, or seventh. This inference plays a crucial role for two reasons:
  • First, because Y is the only wildlife article and must be later than G, consequently only finance and nutrition articles will be among the first three articles edited, and they must conform to the first rule and not have topics that are consecutive;

    Second, because the third rule stipulates that S :longline: Y, S is limited in its positioning. From the setup, S cannot be fifth or seventh, and thus if S were to be later than G, S would have to be sixth. But, if S were sixth, then Y would be seventh, and R would have to be fifth (because G :longline: R). That forces R and S to be consecutive, violating the first rule. Thus, S cannot be fifth, sixth, or seventh, and must then be earlier than G (among the first three).
Because Q cannot be fifth, sixth or seventh, Q must then be among the first three articles edited, which means that both Q and S must be among the first three articles edited. This sets off a chain reaction of inferences:
  • Both Q and S are nutrition articles, and with both among the first three articles while still conforming to the first rule results in the deduction that a nutrition article is first and third, and from the first rule a finance article must be second.

    If S is first, then Q must be third (from the second rule), and if S is third, Q must be first (as determined during the setup), meaning Q and S rotate between the first and third articles.
Let’s take a moment to update the diagram with the inferences made so far:

PT68_Game_#4_#19_diagram 1.png
Because J must be first, second, or third, we can infer that J must be second as that is the only space still available.

H, R, and Y are the three remaining unplaced articles, and while H cannot be fifth due to the first rule, there are no other restrictions on the placement of the variables. This results in the final diagram:

PT68_Game_#4_#19_diagram 2.png
Accordingly, only answer choice (E) could be true, and thus (E) is the correct answer. All in all, a brutal question, especially given that it appears close to the end of the test, when time is typically running down.
You do not have the required permissions to view the files attached to this post.

Get the most out of your LSAT Prep Plus subscription.

Analyze and track your performance with our Testing and Analytics Package.